Med Surg; Chapter 17 - Fluid, Electrolyte, and Acid-Base Imbalances (160 questions)

Ace your homework & exams now with Quizwiz!

The nurse working on a medical unit recognizes that which of the following individuals are a risk for hyponatremia? Select all that apply. 1. A 19-year-old drowning victim rescued from a nearby lake. 2. A 52-year-old with congestive heart failure taking diuretics who is NPO for a cardiac catheterization. 3. A 68-year-old with bowel obstruction receiving nasogastric suction 4. A 92-year-old who is receiving total parenteral nutrition 5. A 55-year-old who takes calcium supplements for osteoporosis 6. A 42-year-old with chronic renal failure.

1, 2, 3, 4

The nurse is caring for a patient with suspected hypokalemia. Which of the following are signs and/or symptoms for which the nurse should be vigilant? Select all that apply. 1. Weak thready pulse 2. Shallow breathing 3. Increased gastrointestinal motility 4. Muscle weakness 5. Nausea 6. Pinpoint pupils

1, 2, 4, 5

The nurse is caring for a patient who is very anxious and hyperventilating. Which of the following are signs and symptoms of respiratory alkalosis for which to observe? Select all that apply. 1. Increased heart rate 2. Increased PaCO2 level 3. Slow deeps respirations 4. Rapid shallow respirations 5. Decreased heart rate 6. Lightheadedness

1, 4, 6

What is an adequate intake of sodium for an adult aged 71 or older? 1. 1.2g 2. 1.3g 3. 1.5g 4. 2.3g

1

Which of the floowing intravenous solutions is isotonic? (select all that apply) 1. 0.9% saline 2. 5% dextrose in water (D5W) 3. 0.45% NaCl 4. Dextrose in 0.255% NaCl

1

The nurse is caring for an older adult patient who presents to the emergency room complaining of severe vomiting and diarrhea, sweating, and rapid heartbeat. The body temperature is normal. Which of the following assessments should the nurse complete next? 1. Evaluate the presence of leg edema 2. Check skin turgor 3. Listen for crackles 4. Assess capillary refill

2

What percentage of an older adult's body weight is water? 1. 30% 2. 50% 3. 60% 4. 70%

2

A positive Chvostek's sign and a positive Trousseau's sign are classic signs of hypocalcemia and of what other electrolyte imbalance? 1. Hypermagnesemia 2. Hyponatremia 3. Hypomagnesemia 4. Hypokalemia

3

Which organ(s) is/are most at risk for dysfunction in a patient with a potassium level of 6.2 mEq/L? 1. Lungs 2. Kidneys 3. Liver 4. Heart

4

A pregnant patient with a family history of cystic fibrosis (CF) asks for information about genetic testing. The most appropriate action by the nurse is to a. refer the patient to a qualified genetic counselor. b. ask the patient why genetic testing is important to her. c. remind the patient that genetic testing has many social implications. d. tell the patient that cystic fibrosis is an autosomal-recessive disorder.

ANS: A A genetic counselor is best qualified to address the multiple issues involved in genetic testing for a patient who is considering having children. Although genetic testing does have social implications, a pregnant patient will be better served by a genetic counselor who will have more expertise in this area. CF is an autosomal-recessive disorder, but the patient might not understand the implications of this statement. Asking why the patient feels genetic testing is important may imply to the patient that the nurse is questioning her value system regarding issues such as abortion.

The home health nurse cares for an alert and oriented older adult patient with a history of dehydration. Which instructions should the nurse give to this patient related to fluid intake? a. "Increase fluids if your mouth feels dry. b. "More fluids are needed if you feel thirsty." c. "Drink more fluids in the late evening hours." d. "If you feel lethargic or confused, you need more to drink."

ANS: A An alert, older patient will be able to self-assess for signs of oral dryness such as thick oral secretions or dry-appearing mucosa. The thirst mechanism decreases with age and is not an accurate indicator of volume depletion. Many older patients prefer to restrict fluids slightly in the evening to improve sleep quality. The patient will not be likely to notice and act appropriately when changes in level of consciousness occur

A postoperative patient who had surgery for a perforated gastric ulcer has been receiving nasogastric suction for 3 days. The patient now has a serum sodium level of 127 mEq/L (127 mmol/L). Which prescribed therapy should the nurse question? a. Infuse 5% dextrose in water at 125 mL/hr. b. Administer IV morphine sulfate 4 mg every 2 hours PRN. c. Give IV metoclopramide (Reglan) 10 mg every 6 hours PRN for nausea. d. Administer 3% saline if serum sodium decreases to less than 128 mEq/L.

ANS: A Because the patient's gastric suction has been depleting electrolytes, the IV solution should include electrolyte replacement. Solutions such as lactated Ringer's solution would usually be ordered for this patient. The other orders are appropriate for a postoperative patient with gastric suction

A patient receives 3% NaCl solution for correction of hyponatremia. Which assessment is most important for the nurse to monitor for while the patient is receiving this infusion? a. Lung sounds b. Urinary output c. Peripheral pulses d. Peripheral edema

ANS: A Hypertonic solutions cause water retention, so the patient should be monitored for symptoms of fluid excess. Crackles in the lungs may indicate the onset of pulmonary edema and are a serious manifestation of fluid excess. Bounding peripheral pulses, peripheral edema, or changes in urine output are also important to monitor when administering hypertonic solutions, but they do not indicate acute respiratory or cardiac decompensation

A patient has a serum calcium level of 7.0 mEq/L. Which assessment finding is most important for the nurse to report to the health care provider? a. The patient is experiencing laryngeal stridor. b. The patient complains of generalized fatigue. c. The patient's bowels have not moved for 4 days. d. The patient has numbness and tingling of the lips.

ANS: A Hypocalcemia can cause laryngeal stridor, which may lead to respiratory arrest. Rapid action is required to correct the patient's calcium level. The other data are also consistent with hypocalcemia, but do not indicate a need for as immediate action as laryngospasm

A patient who has been receiving diuretic therapy is admitted to the emergency department with a serum potassium level of 3.0 mEq/L. The nurse should alert the health care provider immediately that the patient is on which medication? a. Oral digoxin (Lanoxin) 0.25 mg daily b. Ibuprofen (Motrin) 400 mg every 6 hours c. Metoprolol (Lopressor) 12.5 mg orally daily d. Lantus insulin 24 U subcutaneously every evening

ANS: A Hypokalemia increases the risk for digoxin toxicity, which can cause serious dysrhythmias. The nurse will also need to do more assessment regarding the other medications, but they are not of as much concern with the potassium level

A nurse in the outpatient clinic is caring for a patient who has a magnesium level of 1.3 mg/dL. Which assessment would be most important for the nurse to make? a. Daily alcohol intake b. Intake of dietary protein c. Multivitamin/mineral use d. Use of over-the-counter (OTC) laxatives

ANS: A Hypomagnesemia is associated with alcoholism. Protein intake would not have a significant effect on magnesium level. OTC laxatives (such as milk of magnesia) and use of multivitamin/mineral supplements would tend to increase magnesium levels

The nurse is caring for a patient with a massive burn injury and possible hypovolemia. Which assessment data will be of most concern to the nurse? a. Blood pressure is 90/40 mm Hg. b. Urine output is 30 mL over the last hour. c. Oral fluid intake is 100 mL for the last 8 hours. d. There is prolonged skin tenting over the sternum.

ANS: A The blood pressure indicates that the patient may be developing hypovolemic shock as a result of intravascular fluid loss due to the burn injury. This finding will require immediate intervention to prevent the complications associated with systemic hypoperfusion. The poor oral intake, decreased urine output, and skin tenting all indicate the need for increasing the patient's fluid intake but not as urgently as the hypotension

A patient with renal failure has been taking aluminum hydroxide/magnesium hydroxide suspension (Maalox) at home for indigestion. The patient arrives for outpatient hemodialysis and is unresponsive to questions and has decreased deep tendon reflexes. Which action should the dialysis nurse take first? a. Notify the patient's health care provider. b. Obtain an order to draw a potassium level. c. Review the magnesium level on the patient's chart. d. Teach the patient about the risk of magnesium-containing antacids

ANS: A The health care provider should be notified immediately. The patient has a history and manifestations consistent with hypermagnesemia. The nurse should check the chart for a recent serum magnesium level and make sure that blood is sent to the laboratory for immediate electrolyte and chemistry determinations. Dialysis should correct the high magnesium levels. The patient needs teaching about the risks of taking magnesium-containing antacids. Monitoring of potassium levels also is important for patients with renal failure, but the patient's current symptoms are not consistent with hyperkalemia

A patient who is lethargic and exhibits deep, rapid respirations has the following arterial blood gas (ABG) results: pH 7.32, PaO2 88 mm Hg, PaCO2 37 mm Hg, and HCO3 16 mEq/L. How should the nurse interpret these results? a. Metabolic acidosis b. Metabolic alkalosis c. Respiratory acidosis d. Respiratory alkalosis

ANS: A The pH and HCO3 indicate that the patient has a metabolic acidosis. The ABGs are inconsistent with the other responses

A newly admitted patient is diagnosed with hyponatremia. When making room assignments, the charge nurse should take which action? a. Assign the patient to a room near the nurse's station. b. Place the patient in a room nearest to the water fountain. c. Place the patient on telemetry to monitor for peaked T waves. d. Assign the patient to a semi-private room and place an order for a low-salt diet.

ANS: A The patient should be placed near the nurse's station if confused in order for the staff to closely monitor the patient. To help improve serum sodium levels, water intake is restricted. Therefore a confused patient should not be placed near a water fountain. Peaked T waves are a sign of hyperkalemia, not hyponatremia. A confused patient could be distracting and disruptive for another patient in a semiprivate room. This patient needs sodium replacement, not restriction

Following a thyroidectomy, a patient complains of "a tingling feeling around my mouth." Which assessment should the nurse complete immediately? a. Presence of the Chvostek's sign b. Abnormal serum potassium level c. Decreased thyroid hormone level d. Bleeding on the patient's dressing

ANS: A The patient's symptoms indicate possible hypocalcemia, which can occur secondary to parathyroid injury/removal during thyroidectomy. There is no indication of a need to check the potassium level, the thyroid hormone level, or for bleeding

The nurse notes a serum calcium level of 7.9 mg/dL for a patient who has chronic malnutrition. Which action should the nurse take next? a. Monitor ionized calcium level. b. Give oral calcium citrate tablets. c. Check parathyroid hormone level. d. Administer vitamin D supplements.

ANS: A This patient with chronic malnutrition is likely to have a low serum albumin level, which will affect the total serum calcium. A more accurate reflection of calcium balance is the ionized calcium level. Most of the calcium in the blood is bound to protein (primarily albumin). Alterations in serum albumin levels affect the interpretation of total calcium levels. Low albumin levels result in a drop in the total calcium level, although the level of ionized calcium is not affected. The other actions may be needed if the ionized calcium is also decreased

Which action can the registered nurse (RN) who is caring for a critically ill patient with multiple IV lines delegate to an experienced licensed practical/vocational nurse (LPN/LVN)? a. Administer IV antibiotics through the implantable port. b. Monitor the IV sites for redness, swelling, or tenderness. c. Remove the patient's nontunneled subclavian central venous catheter. d. Adjust the flow rate of the 0.9% normal saline in the peripheral IV line.

ANS: B An experienced LPN/LVN has the education, experience, and scope of practice to monitor IV sites for signs of infection. Administration of medications, adjustment of infusion rates, and removal of central catheters in critically ill patients require RN level education and scope of practice

Which of these findings is the best indicator that the fluid resuscitation for a patient with hypovolemic shock has been successful? a. Hemoglobin is within normal limits. b. Urine output is 60 mL over the last hour. c. Pulmonary artery wedge pressure (PAWP) is normal. d. Mean arterial pressure (MAP) is 65 mm Hg.

ANS: B Assessment of end organ perfusion, such as an adequate urine output, is the best indicator that fluid resuscitation has been successful. The hemoglobin level, PAWP, and MAP are useful in determining the effects of fluid administration, but they are not as useful as data indicating good organ perfusion.

A patient comes to the clinic complaining of frequent, watery stools for the last 2 days. Which action should the nurse take first? a. Obtain the baseline weight. b. Check the patient's blood pressure. c. Draw blood for serum electrolyte levels. d. Ask about any extremity numbness or tingling.

ANS: B Because the patient's history suggests that fluid volume deficit may be a problem, assessment for adequate circulation is the highest priority. The other actions are also appropriate, but are not as essential as determining the patient's perfusion status

A patient is admitted for hypovolemia associated with multiple draining wounds. Which assessment would be the most accurate way for the nurse to evaluate fluid balance? a. Skin turgor b. Daily weight c. Presence of edema d. Hourly urine output

ANS: B Daily weight is the most easily obtained and accurate means of assessing volume status. Skin turgor varies considerably with age. Considerable excess fluid volume may be present before fluid moves into the interstitial space and causes edema. Although very important, hourly urine outputs do not take account of fluid intake or of fluid loss through insensible loss, sweating, or loss from the gastrointestinal tract or wounds

When caring for a patient with renal failure on a low phosphate diet, the nurse will inform unlicensed assistive personnel (UAP) to remove which food from the patient's food tray? a. Grape juice b. Milk carton c. Mixed green salad d. Fried chicken breast

ANS: B Foods high in phosphate include milk and other dairy products, so these are restricted on low-phosphate diets. Green, leafy vegetables; high-fat foods; and fruits/juices are not high in phosphate and are not restricted

IV potassium chloride (KCl) 60 mEq is prescribed for treatment of a patient with severe hypokalemia. Which action should the nurse take? a. Administer the KCl as a rapid IV bolus. b. Infuse the KCl at a rate of 10 mEq/hour. c. Only give the KCl through a central venous line. d. Discontinue cardiac monitoring during the infusion.

ANS: B IV KCl is administered at a maximal rate of 10 mEq/hr. Rapid IV infusion of KCl can cause cardiac arrest. Although the preferred concentration for KCl is no more than 40 mEq/L, concentrations up to 80 mEq/L may be used for some patients. KCl can cause inflammation of peripheral veins, but it can be administered by this route. Cardiac monitoring should be continued while patient is receiving potassium because of the risk for dysrhythmias

When assessing a pregnant patient with eclampsia who is receiving IV magnesium sulfate, which finding should the nurse report to the health care provider immediately? a. The bibasilar breath sounds are decreased. b. The patellar and triceps reflexes are absent. c. The patient has been sleeping most of the day. d. The patient reports feeling "sick to my stomach."

ANS: B The loss of the deep tendon reflexes indicates that the patient's magnesium level may be reaching toxic levels. Nausea and lethargy also are side effects associated with magnesium elevation and should be reported, but they are not as significant as the loss of deep tendon reflexes. The decreased breath sounds suggest that the patient needs to cough and deep breathe to prevent atelectasis.

An older adult patient who is malnourished presents to the emergency department with a serum protein level of 5.2 g/dL. The nurse would expect which clinical manifestation? a. Pallor b. Edema c. Confusion d. Restlessness

ANS: B The normal range for total protein is 6.4 to 8.3 g/dL. Low serum protein levels cause a decrease in plasma oncotic pressure and allow fluid to remain in interstitial tissues, causing edema. Confusion, restlessness, and pallor are not associated with low serum protein levels

A patient who had a transverse colectomy for diverticulosis 18 hours ago has nasogastric suction and is complaining of anxiety and incisional pain. The patient's respiratory rate is 32 breaths/minute and the arterial blood gases (ABGs) indicate respiratory alkalosis. Which action should the nurse take first? a. Discontinue the nasogastric suction. b. Give the patient the PRN IV morphine sulfate 4 mg. c. Notify the health care provider about the ABG results. d. Teach the patient how to take slow, deep breaths when anxious.

ANS: B The patient's respiratory alkalosis is caused by the increased respiratory rate associated with pain and anxiety. The nurse's first action should be to medicate the patient for pain. Although the nasogastric suction may contribute to the alkalosis, it is not appropriate to discontinue the tube when the patient needs gastric suction. The health care provider may be notified about the ABGs but is likely to instruct the nurse to medicate for pain. The patient will not be able to take slow, deep breaths when experiencing pain.

The nurse is caring for a patient who has a central venous access device (CVAD). Which action by the nurse is appropriate? a. Avoid using friction when cleaning around the CVAD insertion site. b. Use the push-pause method to flush the CVAD after giving medications. c. Obtain an order from the health care provider to change CVAD dressing. d. Position the patient's face toward the CVAD during injection cap changes.

ANS: B The push-pause enhances the removal of debris from the CVAD lumen and decreases the risk for clotting. To decrease infection risk, friction should be used when cleaning the CVAD insertion site. The dressing should be changed whenever it becomes damp, loose, or visibly soiled. The patient should turn away from the CVAD during cap changes

A patient is admitted to the emergency department with severe fatigue and confusion. Laboratory studies are done. Which laboratory value will require the most immediate action by the nurse? a. Arterial blood pH is 7.32. b. Serum calcium is 18 mg/dL. c. Serum potassium is 5.1 mEq/L. d. Arterial oxygen saturation is 91%.

ANS: B The serum calcium is well above the normal level and puts the patient at risk for cardiac dysrhythmias. The nurse should initiate cardiac monitoring and notify the health care provider. The potassium, oxygen saturation, and pH are also abnormal, and the nurse should notify the health care provider about these values as well, but they are not immediately life threatening

A patient whose mother has been diagnosed with BRCA gene-related breast cancer asks the nurse, "Do you think I should be tested for the gene?" Which response by the nurse is most appropriate? a. "In most cases, breast cancer is not caused by the BRCA gene." b. "It depends on how you will feel if the test is positive for the BRCA gene." c. "There are many things to consider before deciding to have genetic testing." d. "You should decide first whether you are willing to have a double mastectomy."

ANS: C Although presymptomatic testing for genetic disorders allows patients to take action (such as mastectomy) to prevent the development of some genetically caused disorders, patients also need to consider that test results in their medical file may impact insurance, employability, etc. Telling a patient that a decision about mastectomy should be made before testing implies that the nurse has made a judgment about what the patient should do if the test is positive. Although the patient may need to think about her reaction if the test is positive, other issues (e.g., insurance) also should be considered. Although most breast cancers are not related to BRCA gene alterations, the patient with the gene alteration has a markedly increased risk for breast cancer.

The long-term care nurse is evaluating the effectiveness of protein supplements for an older resident who has a low serum total protein level. Which assessment finding indicates that the patient's condition has improved? a. Hematocrit 28% b. Absence of skin tenting c. Decreased peripheral edema d. Blood pressure 110/72 mm Hg

ANS: C Edema is caused by low oncotic pressure in individuals with low serum protein levels. The decrease in edema indicates an improvement in the patient's protein status. Good skin turgor is an indicator of fluid balance, not protein status. A low hematocrit could be caused by poor protein intake. Blood pressure does not provide a useful clinical tool for monitoring protein status

A patient who has a small cell carcinoma of the lung develops syndrome of inappropriate antidiuretic hormone (SIADH). The nurse should notify the health care provider about which assessment finding? a. Reported weight gain b. Serum hematocrit of 42% c. Serum sodium level of 120 mg/dL d. Total urinary output of 280 mL during past 8 hours

ANS: C Hyponatremia is the most important finding to report. SIADH causes water retention and a decrease in serum sodium level. Hyponatremia can cause confusion and other central nervous system effects. A critically low value likely needs to be treated. At least 30 mL/hr of urine output indicates adequate kidney function. The hematocrit level is normal. Weight gain is expected with SIADH because of water retention

A nurse is assessing a newly admitted patient with chronic heart failure who forgot to take prescribed medications and seems confused. The patient complains of "just blowing up" and has peripheral edema and shortness of breath. Which assessment should the nurse complete first? a. Skin turgor b. Heart sounds c. Mental status d. Capillary refill

ANS: C Increases in extracellular fluid (ECF) can lead to swelling of cells in the central nervous system, initially causing confusion, which may progress to coma or seizures. Although skin turgor, capillary refill, and heart sounds also may be affected by increases in ECF, these are signs that do not have as immediate impact on patient outcomes as cerebral edema

A patient has a parenteral nutrition infusion of 25% dextrose. A student nurse asks the nurse why a peripherally inserted central catheter was inserted. Which response by the nurse is most appropriate? a. "There is a decreased risk for infection when 25% dextrose is infused through a central line." b. "The prescribed infusion can be given much more rapidly when the patient has a central line." c. "The 25% dextrose is hypertonic and will be more rapidly diluted when given through a central line." d. "The required blood glucose monitoring is more accurate when samples are obtained from a central line."

ANS: C The 25% dextrose solution is hypertonic. Shrinkage of red blood cells can occur when solutions with dextrose concentrations greater than 10% are administered IV. Blood glucose testing is not more accurate when samples are obtained from a central line. The infection risk is higher with a central catheter than with peripheral IV lines. Hypertonic or concentrated IV solutions are not given rapidly

An older patient receiving iso-osmolar continuous tube feedings develops restlessness, agitation, and weakness. Which laboratory result should the nurse report to the health care provider immediately? a. K+ 3.4 mEq/L (3.4 mmol/L) b. Ca+2 7.8 mg/dL (1.95 mmol/L) c. Na+ 154 mEq/L (154 mmol/L) d. PO4-3 4.8 mg/dL (1.55 mmol/L)

ANS: C The elevated serum sodium level is consistent with the patient's neurologic symptoms and indicates a need for immediate action to prevent further serious complications such as seizures. The potassium and calcium levels vary slightly from normal but do not require immediate action by the nurse. The phosphate level is normal

Which action should the nurse take first when a patient complains of acute chest pain and dyspnea soon after insertion of a centrally inserted IV catheter? a. Notify the health care provider. b. Offer reassurance to the patient. c. Auscultate the patient's breath sounds. d. Give the prescribed PRN morphine sulfate IV.

ANS: C The initial action should be to assess the patient further because the history and symptoms are consistent with several possible complications of central line insertion, including embolism and pneumothorax. The other actions may be appropriate, but further assessment of the patient is needed before notifying the health care provider, offering reassurance, or administration of morphine

After receiving change-of-shift report, which patient should the nurse assess first? a. Patient with serum potassium level of 5.0 mEq/L who is complaining of abdominal cramping b. Patient with serum sodium level of 145 mEq/L who has a dry mouth and is asking for a glass of water c. Patient with serum magnesium level of 1.1 mEq/L who has tremors and hyperactive deep tendon reflexes d. Patient with serum phosphorus level of 4.5 mg/dL who has multiple soft tissue calcium-phosphate precipitates

ANS: C The low magnesium level and neuromuscular irritability suggest that the patient may be at risk for seizures. The other patients have mild electrolyte disturbances and/or symptoms that require action, but they are not at risk for life-threatening complications

The nurse assesses a patient who has been hospitalized for 2 days. The patient has been receiving normal saline IV at 100 mL/hr, has a nasogastric tube to low suction, and is NPO. Which assessment finding would be a priority for the nurse to report to the health care provider? a. Oral temperature of 100.1° F b. Serum sodium level of 138 mEq/L (138 mmol/L) c. Gradually decreasing level of consciousness (LOC) d. Weight gain of 2 pounds (1 kg) above the admission weight

ANS: C The patient's history and change in LOC could be indicative of fluid and electrolyte disturbances: extracellular fluid (ECF) excess, ECF deficit, hyponatremia, hypernatremia, hypokalemia, or metabolic alkalosis. Further diagnostic information is needed to determine the cause of the change in LOC and the appropriate interventions. The weight gain, elevated temperature, crackles, and serum sodium level also will be reported, but do not indicate a need for rapid action to avoid complications

The nurse notes that a patient who was admitted with diabetic ketoacidosis has rapid, deep respirations. Which action should the nurse take? a. Give the prescribed PRN lorazepam (Ativan). b. Start the prescribed PRN oxygen at 2 to 4 L/min. c. Administer the prescribed normal saline bolus and insulin. d. Encourage the patient to take deep, slow breaths with guided imagery.

ANS: C The rapid, deep (Kussmaul) respirations indicate a metabolic acidosis and the need for correction of the acidosis with a saline bolus to prevent hypovolemia followed by insulin administration to allow glucose to reenter the cells. Oxygen therapy is not indicated because there is no indication that the increased respiratory rate is related to hypoxemia. The respiratory pattern is compensatory, and the patient will not be able to slow the respiratory rate. Lorazepam administration will slow the respiratory rate and increase the level of acidosis

A man with mild hemophilia asks the nurse, "Will my children be hemophiliacs?" Which response by the nurse is appropriate? a. "All of your children will be at risk for hemophilia." b. "Hemophilia is a multifactorial inherited condition." c. "Only your male children are at risk for hemophilia." d. "Your female children will be carriers for hemophilia."

ANS: D Because hemophilia is caused by a mutation of the X-chromosome, all female children of a man with hemophilia are carriers of the disorder and can transmit the mutated gene to their offspring. Sons of a man with hemophilia will not have the disorder. Hemophilia is caused by a genetic mutation and is not a multifactorial inherited condition.

Spironolactone (Aldactone), an aldosterone antagonist, is prescribed for a patient. Which statement by the patient indicates that the teaching about this medication has been effective? a. "I will try to drink at least 8 glasses of water every day." b. "I will use a salt substitute to decrease my sodium intake." c. "I will increase my intake of potassium-containing foods." d. "I will drink apple juice instead of orange juice for breakfast."

ANS: D Because spironolactone is a potassium-sparing diuretic, patients should be taught to choose low-potassium foods (e.g., apple juice) rather than foods that have higher levels of potassium (e.g., citrus fruits). Because the patient is using spironolactone as a diuretic, the nurse would not encourage the patient to increase fluid intake. Teach patients to avoid salt substitutes, which are high in potassium

A patient is receiving a 3% saline continuous IV infusion for hyponatremia. Which assessment data will require the most rapid response by the nurse? a. The patient's radial pulse is 105 beats/minute. b. There is sediment and blood in the patient's urine. c. The blood pressure increases from 120/80 to 142/94. d. There are crackles audible throughout both lung fields.

ANS: D Crackles throughout both lungs suggest that the patient may be experiencing pulmonary edema, a life-threatening adverse effect of hypertonic solutions. The increased pulse rate and blood pressure and the appearance of the urine also should be reported, but they are not as dangerous as the presence of fluid in the alveoli

A patient who is taking a potassium-wasting diuretic for treatment of hypertension complains of generalized weakness. It is most appropriate for the nurse to take which action? a. Assess for facial muscle spasms. b. Ask the patient about loose stools. c. Suggest that the patient avoid orange juice with meals. d. Ask the health care provider to order a basic metabolic panel.

ANS: D Generalized weakness is a manifestation of hypokalemia. After the health care provider orders the metabolic panel, the nurse should check the potassium level. Facial muscle spasms might occur with hypocalcemia. Orange juice is high in potassium and would be advisable to drink if the patient was hypokalemic. Loose stools are associated with hyperkalemia

A patient who was involved in a motor vehicle crash has had a tracheostomy placed to allow for continued mechanical ventilation. How should the nurse interpret the following arterial blood gas results: pH 7.48, PaO2 85 mm Hg, PaCO2 32 mm Hg, and HCO3 25 mEq/L? a. Metabolic acidosis b. Metabolic alkalosis c. Respiratory acidosis d. Respiratory alkalosis

ANS: D The pH indicates that the patient has alkalosis and the low PaCO2 indicates a respiratory cause. The other responses are incorrect based on the pH and the normal HCO3

The nurse is caring for a patient who has a calcium level of 12.1 mg/dL. Which nursing action should the nurse include on the care plan? a. Maintain the patient on bed rest. b. Auscultate lung sounds every 4 hours. c. Monitor for Trousseau's and Chvostek's signs. d. Encourage fluid intake up to 4000 mL every day.

ANS: D To decrease the risk for renal calculi, the patient should have a fluid intake of 3000 to 4000 mL daily. Ambulation helps decrease the loss of calcium from bone and is encouraged in patients with hypercalcemia. Trousseau's and Chvostek's signs are monitored when there is a possibility of hypocalcemia. There is no indication that the patient needs frequent assessment of lung sounds, although these would be assessed every shift

What is the nurse's primary concern regarding fluid & electrolytes when caring for an elderly pt who is intermittently confused? 1. risk of dehydration 2. risk of kidney damage 3. risk of stroke 4. risk of bleeding

Answer: 1 Rationale 1: As an adult ages, the thirst mechanism declines. Adding this in a pt with an altered level of consciousness, there is an increased risk of dehydration & high serum osmolality. Rationale 2: The risks for kidney damage are not specifically related to aging or fluid & electrolyte issues. Rationale 3: The risk of stroke is not specifically related to aging or fluid & electrolyte issues. Rationale 4: The risk of bleeding is not specifically related to aging or fluid & electrolyte issues.

The nurse is planning care for a pt with severe burns. Which of the following is this pt at risk for developing? 1. intracellular fluid deficit 2. intracellular fluid overload 3. extracellular fluid deficit 4. interstitial fluid deficit

Answer: 1 Rationale 1: Because this pt was severely burned, the fluid within the cells is diminished, leading to an intracellular fluid deficit. Rationale 2: The intracellular fluid is all fluids that exist within the cell cytoplasm & nucleus. Because this pt was severely burned, the fluid within the cells is diminished, leading to an intracellular fluid deficit. Rationale 3: The extracellular fluid is all fluids that exist outside the cell, including the interstitial fluid between the cells. Because this pt was severely burned, the fluid within the cells is diminished, leading to an intracellular fluid deficit. Rationale 4: The extracellular fluid is all fluids that exist outside the cell, including the interstitial fluid between the cells. Because this pt was severely burned, the fluid within the cells is diminished, leading to an intracellular fluid deficit.

A pt, experiencing multisystem fluid volume deficit, has the symptoms of tachycardia, pale, cool skin, & decreased urine output. The nurse realizes these findings are most likely a direct result of which of the following? 1. the body's natural compensatory mechanisms 2. pharmacological effects of a diuretic 3. effects of rapidly infused intravenous fluids 4. cardiac failure

Answer: 1 Rationale 1: The internal vasoconstrictive compensatory reactions within the body are responsible for the symptoms exhibited. The body naturally attempts to conserve fluid internally specifically for the brain & heart. Rationale 2: A diuretic would cause further fluid loss, & is contraindicated. Rationale 3: Rapidly infused intravenous fluids would not cause a decrease in urine output. Rationale 4: The manifestations reported are not indicative of cardiac failure in this pt.

A pregnant pt is admitted with excessive thirst, increased urination, & has a medical diagnosis of diabetes insipidus. The nurse chooses which of the following nursing diagnoses as most appropriate? 1. Risk for Imbalanced Fluid Volume 2. Excess Fluid Volume 3. Imbalanced Nutrition 4. Ineffective Tissue Perfusion

Answer: 1 Rationale 1: The pt with excessive thirst, increased urination & a medical diagnosis of diabetes insipidus is at risk for Imbalanced Fluid Volume due to the pt &'s excess volume loss that can increase the serum levels of sodium. Rationale 2: Excess Fluid Volume is not an issue for pts with diabetes insipidus, especially during the early stages of treatment. Rationale 3: Imbalanced Nutrition does not apply. Rationale 4: Ineffective Tissue Perfusion does not apply

A pt recovering from surgery has an indwelling urinary catheter. The nurse would contact the pt's primary healthcare provider with which of the following 24-hour urine output volumes? 1. 600 mL 2. 750 mL 3. 1000 mL 4. 1200 mL

Answer: 1 Rationale 1: A urine output of less than 30 mL per hour must be reported to the primary healthcare provider. This indicates inadequate renal perfusion, placing the pt at increased risk for acute renal failure & inadequate tissue perfusion. A minimum of 720 mL over a 24-hour period is desired (30 mL multiplied by 24 hours equals 720 mL per 24 hours).

An elderly postoperative pt is demonstrating lethargy, confusion, & a resp rate of 8 per minute. The nurse sees that the last dose of pain medication administered via a pt controlled anesthesia (PCA) pump was within 30 minutes. Which of the following acid-base disorders might this pt be experiencing? 1. respiratory acidosis 2. metabolic acidosis 3. respiratory alkalosis 4. metabolic alkalosis

Answer: 1 Rationale 1: Acute respiratory acidosis occurs due to a sudden failure of ventilation. Overdoses of narcotic or sedative medications can lead to this condition. Rationale 2: The pt condition being described is respiratory not metabolic in nature. Rationale 3: Acute respiratory acidosis occurs due to a sudden failure of ventilation. Overdoses of narcotic or sedative medications can lead to this condition. Rationale 4: Acute respiratory acidosis occurs due to a sudden failure of ventilation. Overdoses of narcotic or sedative medications can lead to this condition. The pt condition being described is respiratory not metabolic in nature.

A pt is receiving intravenous fluids postoperatively following cardiac surgery. Nursing assessments should focus on which postoperative complication? 1. fluid volume excess 2. fluid volume deficit 3. seizure activity 4. liver failure

Answer: 1 Rationale 1: Antidiuretic hormone & aldosterone levels are commonly increased following the stress response before, during, & immediately after surgery. This increase leads to sodium & water retention. Adding more fluids intravenously can cause a fluid volume excess & stress upon the heart & circulatory system. Rationale 2: Adding more fluids intravenously can cause a fluid volume excess, not fluid volume deficit, & stress upon the heart & circulatory system. Rationale 3: Seizure activity would more commonly be associated with electrolyte imbalances. Rationale 4: Liver failure is not anticipated related to postoperative intravenous fluid administration.

An elderly pt with peripheral neuropathy has been taking magnesium supplements. The nurse realizes that which of the following symptoms can indicate hypomagnesaemia? 1. hypotension, warmth, & sweating 2. nausea & vomiting 3. hyperreflexia 4. excessive urination

Answer: 1 Rationale 1: Elevations in magnesium levels are accompanied by hypotension, warmth, & sweating. Rationale 2: Lower levels of magnesium are associated with nausea & vomiting. Rationale 3: Lower levels of magnesium are associated & hyperreflexia. Rationale 4: Urinary changes are not noted.

A pt is diagnosed with hypokalemia. After reviewing the pt's current medications, which of the following might have contributed to the pt's health problem? 1. corticosteroid 2. thiazide diuretic 3. narcotic 4. muscle relaxer

Answer: 1 Rationale 1: Excess potassium loss through the kidneys is often caused by such meds as corticosteroids, potassium-wasting diuretics, amphotericin B, & large doses of some antibiotics. Rationale 2: Excessive sodium is lost with the use of thiazide diuretics. Rationale 3: Narcotics do not typically affect electrolyte balance. Rationale 4: Muscle relaxants do not typically affect electrolyte balance.

A pt is diagnosed with hyperphosphatemia. The nurse realizes that this pt might also have an imbalance of which of the following electrolytes? 1. calcium 2. sodium 3. potassium 4. chloride

Answer: 1 Rationale 1: Excessive serum phosphate levels cause few specific symptoms. The effects of high serum phosphate levels on nerves & muscles are more likely the result of hypocalcemia that develops secondary to an elevated serum phosphorus level. The phosphate in the serum combines with ionized calcium, & the ionized serum calcium level falls.

A pt prescribed spironolactone is demonstrating ECG changes & complaining of muscle weakness. The nurse realizes this pt is exhibiting signs of which of the following? 1. hyperkalemia 2. hypokalemia 3. hypercalcemia 4. hypocalcemia

Answer: 1 Rationale 1: Hyperkalemia is serum potassium level greater than 5.0 mEq/L. Decreased potassium excretion is seen in potassium-sparing diuretics such as spironolactone. Common manifestations of hyperkalemia are muscle weakness & ECG changes. Rationale 2: Hypokalemia is seen in non-potassium diuretics such as furosemide. Rationale 3: Hypercalcemia has been associated with thiazide diuretics. Rationale 4: Hypocalcemia is seen in pts who have received many units of citrated blood & is not associated with diuretic use.

A pt who is taking digoxin (Lanoxin) is admitted with possible hypokalemia. Which of the following does the nurse realize might occur with this pt? 1. Digoxin toxicity may occur. 2. A higher dose of digoxin (Lanoxin) may be needed. 3. A diuretic may be needed. 4. Fluid volume deficit may occur.

Answer: 1 Rationale 1: Hypokalemia increases the risk of digitalis toxicity in pts who receive this drug for heart failure. Rationale 2: More digoxin is not needed. Rationale 3: A diuretic may cause further fluid loss. Rationale 4: There is inadequate information to assess for concerns related to fluid volume deficits.

The nurse is caring for a pt diagnosed with renal failure. Which of the following does the nurse recognize as compensation for the acid-base disturbance found in pts with renal failure? 1. The pt breathes rapidly to eliminate carbon dioxide. 2. The pt will retain bicarbonate in excess of normal. 3. The pH will decrease from the present value. 4. The pt's oxygen saturation level will improve.

Answer: 1 Rationale 1: In metabolic acidosis compensation is accomplished through increased ventilation or "blowing off" C02. This raises the pH by eliminating the volatile respiratory acid & compensates for the acidosis. Rationale 2: Because compensation must be performed by the system other than the affected system, the pt cannot retain bicarbonate; the manifestation of metabolic acidosis of renal failure is a lower than normal bicarbonate value. Rationale 3: Metabolic acidosis of renal failure causes a low pH; this is the manifestation of the disease process, not the compensation. Rationale 4: Oxygenation disturbance is not part of the acid-base status of the pt with renal failure.

A pt is diagnosed with severe hyponatremia. The nurse realizes this pt will mostly likely need which of the following precautions implemented? 1. seizure 2. infection 3. neutropenic 4. high-risk fall

Answer: 1 Rationale 1: Severe hyponatremia can lead to seizures. Seizure precautions such as a quiet environment, raised side rails, & having an oral airway at the bedside would be included. Rationale 2: Infection precautions not specifically indicated for a pt with hyponatremia. Rationale 3: Neutropenic precautions not specifically indicated for a pt with hyponatremia. Rationale 4: High-risk fall precautions not specifically indicated for a pt with hyponatremia.

When analyzing an arterial blood gas report of a pt with COPD & respiratory acidosis, the nurse anticipates that compensation will develop through which of the following mechanisms? 1. The kidneys retain bicarbonate. 2. The kidneys excrete bicarbonate. 3. The lungs will retain carbon dioxide. 4. The lungs will excrete carbon dioxide.

Answer: 1 Rationale 1: The kidneys will compensate for a respiratory disorder by retaining bicarbonate. Rationale 2: Excreting bicarbonate causes acidosis to develop. Rationale 3: Retaining carbon dioxide causes respiratory acidosis. Rationale 4: Excreting carbon dioxide causes respiratory alkalosis

The nurse is planning care for a pt with fluid volume overload & hyponatremia. Which of the following should be included in this pt's plan of care? 1. Restrict fluids. 2. Administer intravenous fluids. 3. Provide Kayexalate. 4. Administer intravenous normal saline with furosemide.

Answer: 1 Rationale 1: The nursing care for a pt with hyponatremia is dependent on the cause. Restriction of fluids to 1,000 mL/day is usually implemented to assist sodium increase & to prevent the sodium level from dropping further due to dilution. Rationale 2: The administration of intravenous fluids would be indicated in fluid volume deficit & hypernatremia. Rationale 3: Kayexalate is used in pts with hyperkalemia. Rationale 4: The administration of normal saline with furosemide is used to increase calcium secretion.

When caring for a pt diagnosed with hypocalcemia, which of the following should the nurse additionally assess in the pt? 1. other electrolyte disturbances 2. hypertension 3. visual disturbances 4. drug toxicity

Answer: 1 Rationale 1: The pt diagnosed with hypocalcemia may also have high phosphorus or decreased magnesium levels. Rationale 2: The pt with hypocalcemia may exhibit hypotension, & not hypertension. Rationale 3: Visual disturbances do not occur with hypocalcemia. Rationale 4: Hypercalcemia is more commonly caused by drug toxicities.

The nurse is caring for a pt who is anxious & dizzy following a traumatic experience. The arterial blood gas findings include: pH 7.48, PaO2 110, PaCO2 25, & HCO3 24. The nurse would anticipate which initial intervention to correct this problem? 1. Encourage the pt to breathe in & out slowly into a paper bag. 2. Immediately administer oxygen via a mask & monitor oxygen saturation. 3. Prepare to start an intravenous fluid bolus using isotonic fluids. 4. Anticipate the administration of intravenous sodium bicarbonate.

Answer: 1 Rationale 1: This pt is exhibiting signs of hyperventilation that is confirmed with the blood gas results of respiratory alkalosis. Breathing into a paper bag will help the pt to retain carbon dioxide & lower oxygen levels to normal, correcting the cause of the problem. Rationale 2: The oxygen levels are high, so oxygen is not indicated, & would exacerbate the problem if given. Intravenous fluids would not be the initial intervention. Rationale 3: Not enough information is given to determine the need for intravenous fluids. Rationale 4: Bicarbonate would be contraindicated as the pH is already high.

A pt with a history of stomach ulcers is diagnosed with hypophosphatemia. Which of the following interventions should the nurse include in this pt's plan of care? 1. Request a dietitian consult for selecting foods high in phosphorous. 2. Provide aluminum hydroxide antacids as prescribed. 3. Instruct pt to avoid poultry, peanuts, & seeds. 4. Instruct to avoid the intake of sodium phosphate.

Answer: 1 Rationale 1: Treatment of hypophosphatemia includes treating the underlying cause & promoting a high phosphate diet, especially milk, if it is tolerated. Other foods high in phosphate are dried beans & peas, eggs, fish, organ meats, Brazil nuts & peanuts, poultry, seeds & whole grains. Rationale 2: Phosphate-binding antacids, such as aluminum hydroxide, should be avoided. Rationale 3: Poultry, peanuts, & seeds are part of a high phosphate diet. Rationale 4: Mild hypophosphatemia may be corrected by oral supplements, such as sodium phosphate.

The pt has a serum phosphate level of 4.7 mg/dL. Which interdisciplinary treatments would the nurse expect for this pt? Select all that apply. 1. IV normal saline 2. calcium containing antacids 3. IV potassium phosphate 4. encouraging milk intake 5. increasing vitamin D intake

Answer: 1,2 Rationale: Serum phosphate level of 4.7 mg/dL indicates hyperphosphatemia. IV normal saline promotes renal excretion of phosphate.

An elderly pt comes into the clinic with the complaint of watery diarrhea for several days with abdominal & muscle cramping. The nurse realizes that this pt is demonstrating which of the following? 1. hypernatremia 2. hyponatremia 3. fluid volume excess 4. hyperkalemia

Answer: 2 Rationale 1: Hypernatremia is associated with fluid retention & overload. FVE is associated with hypernatremia. Rationale 2: This elderly pt has watery diarrhea, which contributes to the loss of sodium. The abdominal & muscle cramps are manifestations of a low serum sodium level. Rationale 3: This pt is more likely to develop clinical manifestations associated with fluid volume deficit. Rationale 4: Hyperkalemia is associated with cardiac dysrhythmias.

The nurse observes a pt's respirations & notes that the rate is 30 per minute & the respirations are very deep. The metabolic disorder this pt might be demonstrating is which of the following? 1. hypernatremia 2. increasing carbon dioxide in the blood 3. hypertension 4. pain

Answer: 2 Rationale 1: Hypernatremia is associated with profuse sweating & diarrhea. Rationale 2: Acute increases in either carbon dioxide or hydrogen ions in the blood stimulate the respiratory center in the brain. As a result, both the rate & depth of respiration increase. The increased rate & depth of lung ventilation eliminates carbon dioxide from the body, & carbonic acid levels fall, which brings the pH to a more normal range. Rationale 3: The respiratory rate in a pt exhibiting hypertension is not altered. Rationale 4: Pain may be manifested in rapid, shallow respirations.

A pt is prescribed 10 mEq of potassium chloride. The nurse realizes that the reason the pt is receiving this replacement is 1. to sustain respiratory function. 2. to help regulate acid-base balance. 3. to keep a vein open. 4. to encourage urine output.

Answer: 2 Rationale 1: Potassium does not sustain respiratory function. Rationale 2: Electrolytes have many functions. They assist in regulating water balance, help regulate & maintain acid-base balance, contribute to enzyme reactions, & are essential for neuromuscular activity. Rationale 3: Intravenous fluids are used to keep venous access not potassium. Rationale 4: Urinary output is impacted by fluid intake not potassium.

A pt with fluid retention related to renal problems is admitted to the hospital. The nurse realizes that this pt could possibly have which of the following electrolyte imbalances? 1. hypokalemia 2. hypernatremia 3. carbon dioxide 4. magnesium

Answer: 2 Rationale 1: The kidneys are the principal organs involved in the elimination of potassium. Renal failure is often associated with elevations potassium levels. Rationale 2: The kidney is the primary regulator of sodium in the body. Fluid retention is associated with hypernatremia. Rationale 3: Carbon dioxide abnormalities are not normally seen in this type of pt. Rationale 4: Magnesium abnormalities are not normally seen in this type of pt.

The pt, newly diagnosed with diabetes mellitus, is admitted to the emergency department with nausea, vomiting, & abdominal pain. ABG results reveal a pH of 7.2 & a bicarbonate level of 20 mEq/L. Which other assessment findings would the nurse anticipate in this pt? Select all that apply. 1. tachycardia 2. weakness 3. dysrhythmias 4. Kussmaul's respirations 5. cold, clammy skin

Answer: 2,3,4 Rationale: Further assessment findings of this condition are weakness, bradycardia, dysrhythmias, general malaise, decreased level of consciousness, warm flushed skin, & Kussmaul's respirations. Rationale: These ABG results, coupled with the pt's recent diagnosis of diabetes mellitus & history of vomiting would lead the nurse to suspect metabolic acidosis. Further assessment findings of this condition are weakness, bradycardia, dysrhythmias, general malaise, decreased level of consciousness, warm flushed skin, & Kussmaul's respirations.

The pt is receiving intravenous potassium (KCL). Which nursing actions are required? Select all that apply. 1. Administer the dose IV push over 3 minutes. 2. Monitor the injection site for redness. 3. Add the ordered dose to the IV hanging. 4. Use an infusion controller for the IV. 5. Monitor fluid intake & output.

Answer: 2,4,5

An elderly pt does not complain of thirst. What should the nurse do to assess that this pt is not dehydrated? 1. Ask the physician for an order to begin intravenous fluid replacement. 2. Ask the physician to order a chest x-ray. 3. Assess the urine for osmolality. 4. Ask the physician for an order for a brain scan.

Answer: 3 Rationale 1: It is inappropriate to seek an IV at this stage. Rationale 2: There is no indication the pt is experiencing pulmonary complications thus a cheat x-ray is not indicated. Rationale 3: The thirst mechanism declines with aging, which makes older adults more vulnerable to dehydration & hyperosmolality. The nurse should check the pt's urine for osmolality as a 1st step in determining hydration status before other detailed & invasive testing is done. Rationale 4: There is no data to support the need for a brain scan.

A postoperative pt is diagnosed with fluid volume overload. Which of the following should the nurse assess in this pt? 1. poor skin turgor 2. decreased urine output 3. distended neck veins 4. concentrated hemoglobin & hematocrit levels

Answer: 3 Rationale 1: Poor skin turgor is associated with fluid volume deficit. Rationale 2: Decreased urine output is associated with fluid volume deficit. Rationale 3: Circulatory overload causes manifestations such as a full, bounding pulse; distended neck & peripheral veins; increased central venous pressure; cough; dyspnea; orthopnea; rales in the lungs; pulmonary edema; polyuria; ascites; peripheral edema, or if severe, anasarca, in which dilution of plasma by excess fluid causes a decreased hematocrit & blood urea nitrogen (BUN); & possible cerebral edema. Rationale 4: Increased hemoglobin & hematocrit values are associated with fluid volume deficit.

A pt's blood gases show a pH greater of 7.53 & bicarbonate level of 36 mEq/L. The nurse realizes that the acid-base disorder this pt is demonstrating is which of the following? 1. respiratory acidosis 2. metabolic acidosis 3. respiratory alkalosis 4. metabolic alkalosis

Answer: 4 Rationale 1& 2: Respiratory acidosis & metabolic acidosis are both consistent with pH less than 7.35. Rationale 3: Respiratory alkalosis is associated with a pH greater than 7.45 & a PaCO2 of less than 35 mmHG. It is caused by respiratory related conditions. Rationale 4: Arterial blood gases (ABGs) show a pH greater than 7.45 & bicarbonate level greater than 26 mEq/L when the pt is in metabolic alkalosis.

A pt is admitted for treatment of hypercalcemia. The nurse realizes that this pt's intravenous fluids will most likely be which of the following? 1. dextrose 5% & water 2. dextrose 5% & ? normal saline 3. dextrose 5% & ? normal saline 4. normal saline

Answer: 4 Rationale 1: If isotonic saline is not used, the pt is at risk for hyponatremia in addition to the hypercalcemia. Rationale 2: This solution is hypotonic. Isotonic saline is used because sodium excretion is accompanied by calcium excretion through the kidneys. Rationale 3: This solution is hypotonic. Isotonic saline is used because sodium excretion is accompanied by calcium excretion through the kidneys. Rationale 4: Isotonic saline is used because sodium excretion is accompanied by calcium excretion through the kidneys.

The nurse is admitting a pt who was diagnosed with acute renal failure. Which of the following electrolytes will be most affected with this disorder? 1. calcium 2. magnesium 3. phosphorous 4. potassium

Answer: 4 Rationale 1: This pt will be less likely to develop a calcium imbalance. Rationale 2: This pt will be less likely to develop a magnesium imbalance. Rationale 3: This pt will be less likely to develop a phosphorous imbalance. Rationale 4: Because the kidneys are the principal organs involved in the elimination of potassium, renal failure

A 28-year-old male pt is admitted with diabetic ketoacidosis. The nurse realizes that this pt will have a need for which of the following electrolytes? 1. sodium 2. potassium 3. calcium 4. magnesium

Answer: 4 Rationale 4: One risk factor for hypomagnesaemia is an endocrine disorder, including diabetic ketoacidosis.

An elderly pt who is being medicated for pain had an episode of incontinence. The nurse realizes that this pt is at risk for developing 1. dehydration. 2. over-hydration. 3. fecal incontinence. 4. a stroke.

Correct Answer: 1 Rationale 1: Functional changes of aging also affect fluid balance. Older adults who have self-care deficits, or who are confused, depressed, tube-fed, on bed rest, or taking medications (such as sedatives, tranquilizers, diuretics, & laxatives), are at greatest risk for fluid volume imbalance. Rationale 2: There is inadequate evidence to support the risk of over-hydration. Rationale 3: There is inadequate evidence to support the risk of fecal incontinence. Rationale 4: There is inadequate evidence to support the risk of a stroke.

Which pts are at risk for the development of hypercalcemia? Select all that apply. 1. the pt with a malignancy 2. the pt taking lithium 3. the pt who uses sunscreen to excess 4. the pt with hyperparathyroidism 5. the pt who overuses antacids

Correct Answer: 1,2,4,5 Rationale 1: Pts with malignancy are at risk for development of hypercalcemia due to destruction of bone or the production of hormone-like substances by the malignancy. Rationale 2: Lithium & overuse of antacids can result in hypercalcemia. Hypercalcemia can result from hyperparathyroidism which causes release of calcium from the bones, increased calcium absorption in the intestines & retention of calcium by the kidneys. Rationale 3: The pt who uses sunscreen to excess is more likely to have a vitamin D deficiency which would result in hypocalcemia. Rationale 4: Hypercalcemia can result from hyperparathyroidism which causes release of calcium from the bones, increased calcium absorption in the intestines & retention of calcium by the kidneys. Rationale 5: Lithium & overuse of antacids can result in hypercalcemia.

A pt is admitted with burns over 50% of his body. The nurse realizes that this pt is at risk for which of the following electrolyte imbalances? 1. hypercalcemia 2. hypophosphatemia 3. hypernatremia 4. hypermagnesemia

Correct Answer: 2 Rationale 1: Pts who experience burns are not at an increased risk for developing increased blood calcium levels. Rationale 2: Causes of hypophosphatemia include stress responses & extensive burns. Rationale 3: Pts who experience burns are not at an increased risk for developing increased blood sodium levels. Rationale 4: Pts who experience burns are not at an increased risk for developing increased blood magnesium levels.

Which pH value respresents acidosis? a. 7.26 b. 7.35 c. 7.4 d. 7.49

a

An older woman was admitted to the medical unit with dehydration. Clinical indications of this problem are (select all that apply): a. weight loss b. dry oral mucosa c. full bounding pulse d. engorged neck veins e. decreased central venous pressure

a, b, & e Rationale: Body weight loss, especially sudden change, is an excellent indicator of overall fluid volume loss. Other clinical manifestations of dehydration include dry mucous membranes and a decreased central venous pressure, which reflect fluid volume loss.

The dehydrated patient is receiving a hypertonic solution. What assessments must be done to avoid risk factors of these solutions (select all that apply)? a. Lung sounds b. Bowel sounds c. Blood pressure d. Serum sodium level e. Serum potassium level

a, c, & d BP, lung sounds, and serum sodium levels must be monitored frequently because of the risk for excess intravascular volume with hypertonic solutions.

You are caring for a patient admitted with diabetes mellitus, malnutrition, and massive GI bleed. In analyzing the morning lab results, the nurse understands that a potassium level of 5.5 mEq/L could be caused by which factors in this patient (select all that apply)? a. The potassium level may be increased if the patient has renal nephropathy. b. The patient may be excreting extra sodium and retaining potassium because of malnutrition. c. The potassium level may be increased as a result of dehydration that accompanies high blood glucose levels. d. There may be excess potassium being released into the blood as a result of massive transfusion of stored hemolyzed blood. e. The patient has been overeating raisins, baked beans, and salt substitute that increase the potassium level.

a, c, & d Hyperkalemia may result from hyperglycemia, renal insufficiency, and/or cell death. Diabetes mellitus, along with the stress of hospitalization and illness, can lead to hyperglycemia. Renal insufficiency is a complication of diabetes. Malnutrition does not cause sodium excretion accompanied by potassium retention. Thus it is not a contributing factor to this patient's potassium level. Stored hemolyzed blood can cause hyperkalemia when large amounts are transfused rapidly. The patient with a massive GI bleed would have an NG tube and not be eating.

It is especially important for the nurse to assess for which clinical manifestation(s) in a patient who has just undergone a total thyroidectomy (select all that apply)? a. Confusion b, Weight gain c. Depressed reflexes d. Circumoral numbness e. Positive Chxostek's sign

a, d, & e Rationale: Inadvertent removal of a portion of or injury to the parathyroid glands during thyroid or neck surgery can result in a lack of parathyroid hormone, leading to hypocalcemia. A positive Chvostek sign, confusion, and circumoral numbness are manifestations of low serum calcium levels.

Which of the following are functions of sodium in the body? Select all that apply. a. Maintenance of serum osmolarity b. Formation of bones and teeth c. Control of bronchodilation d. Control of serum glucose e. Maintenance of cellular function

a, e Sodium is the major cation in the blood and helps maintain serum osmolarity. Sodium is also important for cell function, especially in the central nervous system.

The home health nurse cares for an alert and oriented older adult patient with a history of dehydration. Which instructions should the nurse give to this patient related to fluid intake? a. "Increase fluids if your mouth feels dry. b. "More fluids are needed if you feel thirsty." c. "Drink more fluids in the late evening hours." d. "If you feel lethargic or confused, you need more to drink."

a. "Increase fluids if your mouth feels dry.

Which serum potassium result best supports the rationale for administering a stat dose of potassium chloride 20 mEq in 250 mL of normal saline over 2 hours? a. 3.1 mEq/L b. 3.9 mEq/L c. 4.6 mEq/L d. 5.3 mEq/L

a. 3.1 mEq/L The normal range for serum potassium is 3.5 to 5.0 mEq/L. This IV order provides a substantial amount of potassium. Thus the patient's potassium level must be low. The only low value shown is 3.1 mEq/L.

When assessing the patient with a multi-lumen central line, the nurse notices that the cap is off one of the lines. On assessment, the patient is in respiratory distress, and the vital signs show hypotension and tachycardia. What is the nurse's priority action? a. Administer oxygen. b. Notify the physician. c. Rapidly administer more IV fluid. d. Reposition the patient to the right side.

a. Administer oxygen. The cap off the central line could allow entry of air into the circulation. For an air emboli, oxygen is administered; the catheter is clamped; the patient is positioned on the left side with the head down. Then the physician is notified.

A young couple comes to the obstetrician's office prior to attempting to conceive. They have concerns regarding possible genetic defects from a long family history in both families. Which of the following topics should the nurse explore with the couple in order to prepare them for the genetic evaluation? a. Communication should include an assessment of the positive and negative outcomes of the test b. All genetic testing, depending on the results, does not need to remain confidential. c. Informed consent would be nice to have but is not mandatory. d. All genetic testing falls into voluntary and involuntary categories.

a. Communication should include an assessment of the positive and negative outcomes of the test Rationale: With knowledge of available genetic tests and the many implications related to genetic testing, the nurse can assist clients as they weigh choices regarding genetic testing #2 is incorrect because clients should engage in genetic testing with full knowledge, confidentiality and act autonomously. # 3 is incorrect because informed consent in needed in order to do the testing. #4 is incorrect because all genetic testing should be voluntary.

A nurse in the outpatient clinic is caring for a patient who has a magnesium level of 1.3 mg/dL. Which assessment would be most important for the nurse to make? a. Daily alcohol intake b. Intake of dietary protein c. Multivitamin/mineral use d. Use of over-the-counter (OTC) laxatives

a. Daily alcohol intake

A patient is being discharged following hospitalization for fluid imbalance. Which instruction by the nurse should take priority? a. "Weigh yourself at the same time every day and report changes." b. "Call your doctor immediately if you feel weak or fatigued." c. "Drink eight glasses of water a day." d. "Measure everything you drink, and measure how much you urinate each day."

a. Daily or every-other-day weights are easy to keep track of at home.

Measurements related to fluid balance of clients that a nurse can initiate without a physician's order include: a. Daily weights, vital signs, and fluid intake and output b. Daily weights, diuretics, and waist measurement c. Monitoring temperature, fluid intake and output, and calorie count d. Auscultating lung sounds, monitoring color of urine, and placing a Foley catheter into the client

a. Daily weights, vital signs, and fluid intake and output Rationale: Daily weights, checking vital signs, and monitoring fluid I&O all fall within the realm of nursing interventions. The remaining interventions either have the nurse perform a task requiring an MD order, such as giving diuretics or placing a Foley catheter, or have an action unrelated to this problem, such as the calorie count.

When caring for a patient with fluid excess, which of the following interventions will best help relieve respiratory distress? a. Elevate the head of the bed b. Encourage the patient to cough and deep breathe. c. Increase fluids to promote urine output. d. Perform percussion and postural drainage.

a. Elevating the head of the bed will provide more room for lung expansion and provide the quickest relief for shortness of breath.

A patient who has been receiving diuretic therapy is admitted to the emergency department with a serum potassium level of 3.0 mEq/L. The nurse should alert the health care provider immediately that the patient is on which medication? a. Oral digoxin (Lanoxin) 0.25 mg daily b. Ibuprofen (Motrin) 400 mg every 6 hours c. Metoprolol (Lopressor) 12.5 mg orally daily d. Lantus insulin 24 U subcutaneously every evening

a. Oral digoxin (Lanoxin) 0.25 mg daily

Following a thyroidectomy, a patient complains of "a tingling feeling around my mouth." Which assessment should the nurse complete immediately? a. Presence of the Chvostek's sign b. Abnormal serum potassium level c. Decreased thyroid hormone level d. Bleeding on the patient's dressing

a. Presence of the Chvostek's sign

Edema that forms in clients with kidney disease is due to: a. Reduced plasma oncotic pressure, so that fluid is not drawn into the capillaries from interstitial tissues b. Decreased capillary hydrostatic pressures pushing fluid into the interstitial tissues c. Capillaries becoming less permeable, allowing fluid to escape into interstitial tissues d. Obstructed lymph flow that assists the movement of fluid from the interstitial tissues back into the vascular compartment

a. Reduced plasma oncotic pressure, so that fluid is not drawn into the capillaries from interstitial tissues Rationale: The edema is due to low levels of plasma proteins that exist with this disease, altering the oncotic pressure that helps regulate fluid movement in the vascular space moving into interstitial area. Increased capillary hydrostatic pressure is the cause. Capillaries have increased permeability when edema formation is possible. Obstructed lymph flow impairs the movement of fluid from interstitial tissues back into the vascular compartment, resulting in edema.

The patient has chronic kidney disease and ate a lot of nuts, bananas, peanut butter, and chocolate. The patient is admitted with loss of deep tendon reflexes, somnolence, and altered respiratory status. What treatment should the nurse expect for this patient? a. Renal dialysis b. IV potassium chloride c. IV furosemide (Lasix) d. IV normal saline at 250 mL per hour

a. Renal dialysis Renal dialysis will need to be administered to remove the excess magnesium that is in the blood from the increased intake of foods high in magnesium. If renal function was adequate, IV potassium chloride would oppose the effects of magnesium on the cardiac muscle. IV furosemide and increased fluid would increase urinary output which is the major route of excretion for magnesium.

A client suffering from a narcotic overdose is seen in the Emergency Department. The client is confused, with warm, flushed skin, headache, and weakness. Vital signs of noted are T 102.6, HR 128, R 24, and BP 130/86. A blood gas analysis sample was drawn on room air, and the results are as follows: pH 7.33, PaCO2 53, PaO2 72, HCO3 24. This client is at risk for: a. Respiratory acidosis b. Respiratory alkalosis c. Metabolic acidosis d. Metabolic alkalosis

a. Respiratory acidosis Rationale: Narcotic overdose causes more carbonic acid levels to rise because of hypoventilation and carbon dioxode retention.

You receive a physician's order to change a patient's IV from D5½ NS with 40 mEq KCl/L to D5NS with 20 mEq KCl/L. Which serum laboratory values on this same patient best support the rationale for this IV order change? a. Sodium 136 mEq/L, potassium 4.5 mEq/L b. Sodium 145 mEq/L, potassium 4.8 mEq/L c. Sodium 135 mEq/L, potassium 3.6 mEq/L d. Sodium 144 mEq/L, potassium 3.7 mEq/L

a. Sodium 136 mEq/L, potassium 4.5 mEq/L The normal range for serum sodium is 135 to 145 mEq/L, and the normal range for potassium is 3.5 to 5.0 mEq/L. The change in the IV order decreases the amount of potassium and increases the amount of sodium. Therefore for this order to be appropriate, the potassium level must be near the high end and the sodium level near the low end of their respective ranges.

A couple is undergoing prenatal genetic counseling. The nurse is preparing to deliver genetic education and counseling. What is the best approach for the nurse to take? a. The nurse should discuss the positives and negatives of each decision and present as many options as possible through the use of therapeutic listening and communication skills. b. The nurse should provide genetic counseling in a direct, non threatening way. c. The nurse should attempt to influence the decision of the client because the nurse has more knowledge in this area. d. The nurse should withhold any bad news from the client until the time is right.

a. The nurse should discuss the positives and negatives of each decision and present as many options as possible through the use of therapeutic listening and communication skills. Rationale: Genetic healthcare providers present the client with information to promote informed decisions. # 1 is a correct statement. # 2 is incorrect because the counseling should be provided with a non-directional approach. # 3 is incorrect because clients should be permitted to make decisions that are not influenced by any biases or values from the nurse. # 4 is incorrect because the nurse should not withhold any information from the client regardless of what it is.

While caring for a patient with metastatic bone cancer, which clinical manifestations would alert the nurse to the possibility of hypercalcemia in this patient? a. Weakness b. Paresthesia c. Facial spasms d. Muscle tremors

a. Weakness Signs of hypercalcemia are lethargy, headache, weakness, muscle flaccidity, heart block, anorexia, nausea, and vomiting. Paresthesia, facial spasms, and muscle tremors are symptoms of hypocalcemia.

The nurse is preparing to administer the discharge teaching instructions to a client with heart failure. Which measure will be included that the client will need to monitor on their own related to their recent hospitalization: _________________ . a. daily weight b. weekly electrolyte level c. daily fasting blood sugar d. daily urine dipstick for protein

a. daily weight Daily weights are one of the most important determinations of fluid balance. Weighing at home at the same time with the same amount of clothing on will track overall fluid changes as they occur. Acute weight gain or loss represents fluid gain or loss. The other measures would not be the responsibility of the client. Blood sugar measurement does not directly apply in this situation. .

The nursing care for a patient with hyponatremia includes: a. fluid restriction b. administration of hypotonic IV fluids c. administration of a cation-exchance resin d. increased water intake for patients on nasogastric suction

a. fluid restriction Rationale: In hyponatremia that is caused by water excess, fluid restriction often is all that is needed to treat the problem.

The lungs act as an acid-base buffer by: a. increasing respiratory rate and depth when CO2 levels in the blood are high, reducing acid load. b. increasing respiratory rate and depth when CO2 levels in the blood are low, reducing base load c. decreasing respiratory rate and depth when CO2 levels in the blood are high, reducing acid load. d. decreasing respiratory rate and depth when CO2 levels in the blood are low, increasing acid load.

a. increasing respiratory rate and depth when CO2 levels in the blood are high, reducing acid load. Rationale: As a compensatory mechanism, the respiratory system acts on the CO2 + H2O side of the reaction by altering the rate and depth of breathing to "blow off" (through hyperventilation) or "retain" (through hypoventilation) CO2.

During administration of a hypertonic IV solution, the mechanism involved in equalizing the fluid concentration between ECF and the cells is: a. osmosis b. diffusion c. active transport d. facilitated diffusion

a. osmosis Rationale: Osmosis is the movement of water between two compartments separated by a semipermeable membrane. Water moves through the membrane from an area of low solute concentration to an area of high solute concentration.

You are caring for a patient admitted with an exacerbation of asthma. After several treatments, the ABG results are pH 7.40, PaCO2 40 mm Hg, HCO3 24 mEq/L, PaO2 92 mm Hg, and O2 saturation of 99%. You interpret these results as a. within normal limits. b. slight metabolic acidosis. c. slight respiratory acidosis. d. slight respiratory alkalosis.

a. within normal limits. The normal pH is 7.35 to 7.45. Normal PaCO2 levels are 35 to 45 mm Hg, and HCO3 is 22 to 26 mEq/L. Normal PaO2 is >80 mm Hg. Normal oxygen saturation is >95%. Since the patient's results all fall within these normal ranges, the nurse can conclude that the patient's blood gas results are within normal limits.

Which patient is at risk for respiratory acidosis? a. The patient with uncontrolled diabetes mellitus b. The patient with chronic pulmonary disease c. The patient who is very anxious d. The patient who overuses antacids

b

A 93-year-old patient with diarrhea and dehydration is admitted to the hospital from an extended care facility. For which of the following symptoms of dehydration should the nurse assess? a. Pale-colored urine, bradycardia b. Disorientation, poor skin turgor c. Decreased hematocrit, hypothermia d. Lung congestion, abdominal discomfort

b Dehydration is associated with poor skin turgor because of loss of water in the tissues, and with disorientation because of loss of blood volume in the brain.

While performing patient teaching regarding hypercalcemia, which statements are appropriate (select all that apply)? a. Have patient restrict fluid intake to less than 2000 mL/day. b. Renal calculi may occur as a complication of hypercalcemia. c. Weight-bearing exercises can help keep calcium in the bones. d. The patient should increase daily fluid intake to 3000 to 4000 mL. e. Treatment of heartburn can best be managed with Tums as needed.

b, c, & d A daily fluid intake of 3000 to 4000 mL is necessary to enhance calcium excretion and prevent the formation of renal calculi, a potential complication of hypercalcemia. Tums are a calcium-based antacid that should not be used in patients with hypercalcemia. Weight-bearing exercise does enhance bone mineralization.

A patient is being placed on a potassium-losing diuretic. Which foods are high in potassium and should be recommended to the patient by the nurse? Select all that apply. a. Bread b. Potato c. Tomato juice d. Banana e. Gelatin

b, c, d Potatoes, tomatoes and bananas are highest in potassium.

Which question by a client hospitalized for a fluid volume deficit would require the nurse to investigate in more detail the probable cause of the dehydration? a. "Do I have to drink everything that is sent on my meal trays? I do not drink that much at home." b. "I have not had a bowel movement for 2 days. Can I get a laxative?" c. "I have tried to limit my sodium intake at home, but I am not very successful. Will I have to continue with this?" d. "I try to use the bathroom many times a day so that I will not have to get up through the night. Do you think this caused me to lose so much water?"

b. "I have not had a bowel movement for 2 days. Can I get a laxative?" Chronic abuse of laxatives and/or enemas may lead to fluid loss in a client. Elderly clients are most at risk for this as their overall fluid composition to total body weight has decreased. The frequent or dependent use of laxatives or enemas may not be readily reported on admission.

A client is brought to the emergency room following a motor vehicle accident. There is a moderate amount of blood loss. The physician has ordered 1000 ml of intravenous fluids to infuse over 4 hours. Which type of solution would be appropriate? Select all that apply. a. 5% dextrose with 0.45% sodium chloride b. 0.9% sodium chloride c. 5% dextrose in water d. 0.45% sodium chloride

b. 0.9% sodium chloride d. 0.45% sodium chloride Isotonic solutions (similar to normal body fluid) are used to expand plasma volume in hypotensive clients or to replace abnormal losses. Fluid loss due to injury will usually cause isotonic fluid loss. Expanding plasma volume will increase the overall circulating body volume. 5% dextrose with 0.45% sodium chloride is a hypotonic solution and is used when electrolyte replacement is also needed.

The patient was admitted for a paracentesis to remove ascites fluid. Five liters of fluid was removed. What IV solution may be used to pull fluid into the intravascular space after the paracentesis? a. 0.9% sodium chloride b. 25% albumin solution c. Lactated Ringer's solution d. 5% dextrose in 0.45% saline

b. 25% albumin solution After a paracentesis of 5 L or greater of ascites fluid, 25% albumin solution may be used as a volume expander. Normal saline, lactated Ringer's, and 5% dextrose in 0.45% saline will not be effective for this action.

Which of the following hospitalized clients would the nurse be most concerned is at risk for developing an imbalance related to water loss? Select all that apply. a. A 50-year old undernourished female b. A 75-year old female of average body weight c. A 60-year old male of average body weight d. A 45-year old obese male

b. A 75-year old female of average body weight d. A 45-year old obese male On average, females have a lower proportion of water to total body weight composition. The obese client has a lower proportion of water to total body weight. After age 65, total body water may decrease 45-50% of the total body weight.

The nurse working in a clinic determines that which of the following clients should be referred for genetic counseling? a. A woman with no prior history of breast cancer. b. A man with a family history of prostate cancer. c. A man without a mutation in MLH1/MSH2. d. A woman without a mutation in BRCA1 and BRCA2..

b. A man with a family history of prostate cancer. Rationale: With knowledge of genetic conditions, the nurse can ensure health teaching and early detection of complications from genetic conditions with emphasis on primary and secondary care interventions. Without a previous family history or gene mutation a client does not need to be referred to a genetic professional. #1 the client has no prior history. # 2 the client has a family history so he is an appropriate referral. # 3 the client does not need to be referred because he has no gene mutation suggesting colorectal cancer. # 4 the client does not need a referral because she does not have a mutation suggestive of breast cancer.

Which individual would least likely suffer from a disturbance in fluid volume, electrolyte, or acid-base balance? a. An infant suffering from gastroenteritis for three days b. An elderly client suffering from a type I decubitus c. Adults with impaired cardiac function d. Clients who are confused

b. An elderly client suffering from a type I decubitus Rationale: The proportion of body water decreases with aging. Tissue trauma, such as burns, causes fluids and electrolytes to be lost from the damaged cells, and the breakdown in the continuity of the tissue. In Type I Decubitus, the skin remains intact, and any shifting of fluids is due to the inflammatory process and internally maintained within the body. Vomiting and diarrhea can cause significant fluid loses. Age, sex, and body fat affect total body water. Infants have the delete spaces highest proportion of water; it accounts for 70-80% of their body weight. Decreased blood flow to the kidneys as caused by impaired cardiac function stimulates the renin-angiotensin-aldosterone system, causing sodium and water retention. Clients who are confused or unable to communicate are at risk for inadequate fluid intake. Age does not play a significant factor here.

While orienting a new nurse to work in the surgical intensive care unit, the charge nurse quizzes the new nurse as to which of the following postoperative clients would be at highest risk for problems related to excess fluids? Select all that apply. a. Client with diabetes insipidus b. Client with heart failure c. Client with systemic lupus erythematous d. Client with liver cirrhosis

b. Client with heart failure d. Client with liver cirrhosis With fluid volume excess, water and sodium are gained together. This creates a total water increase in the body. A client with a heart that is not able to pump this excess fluid and a liver that is not functioning appropriately will have potential problems. Clients with diabetes insipidus and systemic lupus erythematous are not considered to be at general risk for fluid volume excess. .

A patient is admitted for hypovolemia associated with multiple draining wounds. Which assessment would be the most accurate way for the nurse to evaluate fluid balance? a. Skin turgor b. Daily weight c. Presence of edema d. Hourly urine output

b. Daily weight

An elderly patient was hydrated with lactated Ringer's solution in the emergency room for the last hour. During the most recent evaluation of the patient by the nurse, a finding of a rapid bounding pulse and shortness of breath were noted. Reporting this episode to the physician, the nurse suspects that the patient now shows signs of: a. Hypovolemia, and needs more fluids b. Hypervolemia, and needs the fluids adjusted c. An acid-base disturbance d. Needing no adjustment in fluid administration

b. Hypervolemia, and needs the fluids adjusted Rationale: Isotonic solutions has the same osmolality as body fluids. Isotonic solutions, such as Normal Saline and Ringer's Lactate, initially remain in the vascular compartment, expanding vascular volume. Isotonic imbalances occur when water and electrolytes are lost or gained in equal proportions, and serum osmolality remains constant.

IV potassium chloride (KCl) 60 mEq is prescribed for treatment of a patient with severe hypokalemia. Which action should the nurse take? a. Administer the KCl as a rapid IV bolus. b. Infuse the KCl at a rate of 10 mEq/hour. c. Only give the KCl through a central venous line. d. Discontinue cardiac monitoring during the infusion.

b. Infuse the KCl at a rate of 10 mEq/hour.

The patient is admitted with metabolic acidosis. Which system is not functioning normally? a. Buffer system b. Kidney system c. Hormone system d. Respiratory system

b. Kidney system When the patient has metabolic acidosis, the kidneys are not combining H+ with ammonia to form ammonium or eliminating acid with secretion of free hydrogen into the renal tubule. The buffer system neutralizes hydrochloric acid by forming a weak acid. The hormone system is not directly related to acid-base balance. The respiratory system releases CO2 that combines with water to form hydrogen ions and bicarbonate. The hydrogen is then buffered by the hemoglobin.

You are caring for a patient admitted with heart failure. The morning laboratory results reveal a serum potassium level of 2.9 mEq/L. Which classification of medications should you withhold until consulting with the physician? a. Antibiotics b. Loop diuretics c. Bronchodilators d. Antihypertensives

b. Loop diuretics Loop diuretics are contraindicated during episodes of hypokalemia because these medications cause the kidneys to excrete sodium and potassium. Thus administration of this type of medication at this time would worsen the hypokalemia, putting the patient at risk for dysrhythmias. The prescribing physician should be consulted for potassium replacement therapy, and the drug should be withheld until the potassium has returned to normal range.

The ambulance arrives with a client who presents with Kussmaul's respirations. The client has a history of diabetes. The nurse does a fingerstick blood sugar test immediately. The nurse is anticipating results that will lead to the diagnosis of which acid-base imbalance: a. Respiratory acidosis. b. Metabolic acidosis. c. Respiratory alkalosis. d. Metabolic alkalosis.

b. Metabolic acidosis. Kussmaul's respirations are deep and rapid respirations that are a compensatory mechanism by the respiratory system to return the blood pH to normal by eliminating carbon dioxide. This occurs when the body is in a metabolic acidotic state. The nurse expects the blood sugar to be very high. When the blood sugar of a diabetic client is very high, diabetic ketoacidosis may result. This is a type of metabolic acidosis. Clients with a history of diabetes are not prone to respiratory imbalances.

You are admitting a patient with complaints of abdominal pain, nausea, and vomiting. A bowel obstruction is suspected. You assess this patient for which anticipated primary acid-base imbalance if the obstruction is high in the intestine? a. Metabolic acidosis b. Metabolic alkalosis c. Respiratory acidosis d. Respiratory alkalosis

b. Metabolic alkalosis Because gastric secretions are rich in hydrochloric acid, the patient who is vomiting will lose a significant amount of gastric acid and be at an increased risk for metabolic alkalosis.

Which nursing intervention is most appropriate when caring for a patient with dehydration? a. Auscultate lung sounds every 2 hours. b. Monitor daily weight and intake and output. c. Monitor diastolic blood pressure for increases. d. Encourage the patient to reduce sodium intake.

b. Monitor daily weight and intake and output. Measuring weight is the most reliable means of detecting changes in fluid balance. Weight loss would indicate the dehydration is worsening, whereas weight gain would indicate restoration of fluid volume. Recall that a 1-kg weight gain indicates a gain of approximately 1000 mL of body water.

You are caring for a patient admitted with a diagnosis of chronic obstructive pulmonary disease (COPD) who has the following arterial blood gas results: pH 7.33, PaO2 47 mm Hg, PaCO2 60 mm Hg, HCO3 32 mEq/L, and O2 saturation of 92%. What is the correct interpretation of these results? a. Fully compensated respiratory alkalosis b. Partially compensated respiratory acidosis c. Normal acid-base balance with hypoxemia d. Normal acid-base balance with hypercapnia

b. Partially compensated respiratory acidosis A low pH (normal 7.35-7.45) indicates acidosis. In the patient with respiratory disease such as COPD, the patient retains carbon dioxide (normal 35-45 mm Hg), which acts as an acid in the body. For this reason, the patient has respiratory acidosis. The elevated HCO3 indicates a partial compensation for the elevated CO2.

The client with a history of chronic obstructive pulmonary disease has 10 liters of oxygen per nasal cannula applied by the student nurse for complaints of shortness of breath. The nursing instructor reduces this to 2 liters after reading the student's documentation. What is the rationale for this intervention? a. The student did not evaluate the client with the nursing instructor. b. The client's history prohibits the use of high levels of supplemental oxygen. c. The physician did not order the oxygen. d. The client needs to be evaluated by respiratory therapy before supplemental oxygen can be applied.

b. The client's history prohibits the use of high levels of supplemental oxygen. Clients with chronic lung disease may have consistently high carbon dioxide levels in their blood. Administering a high level of oxygen to these clients could inhibit their drive to breath as their carbon dioxide levels primarily determine respiratory response. ."

The nurse anticipates treatment of the patient with hyperphosphatemia secondary to renal failure will include: a. fluid restriction b. calcium supplements c. loop diuretic therapy d. magnesium supplements

b. calcium supplements Rationale: The major conditions that can lead to hyperphosphatemia are acute kidney injury and chronic kidney disease that alter the ability of the kidneys to excrete phosphate. For the patient with renal failure, measures to reduce serum phosphate levels include calcium supplements, phosphate-binding agents or gels, fluid replacement therapy, and dietary phosphate restrictions.

A client is admitted to the hospital for dehydration related to a gastrointestinal viral illness. The client is also on an alcohol withdrawal protocol. They complain of their lips and mouth feeling numb and tingling. When the nurse observes the lab results from the morning, which result would provide an insight into this client's complaint: a. low sodium level. b. low calcium level. c. high magnesium level. d. high potassium level.

b. low calcium level. Clients with a history of alcoholism are prone to hypocalcemia. The numbness and tingling of the lips and mouth are symptomatic of this and are referred to as "tetany."

A client recovering from a spinal cord injury becomes angry with the nurse and uses obscenity. The nurse's best response is: a. To laugh in order to relieve the tension b. "Stop it right now. This is uncalled for." c. "I'm listening. Tell me what this is about." d. "What did I do to make you so angry?"

c. "I'm listening. Tell me what this is about." Rationale: Clients with trauma have a need to express anger and have it acknowledged. Laughing does not acknowledge the client's need to express strong displeasure with a severe injury. The client's anger is best acknowledged in such a manner that the client does not suppress future attempts to express emotion. The client owns the anger, and needs to assume responsibility for the anger and deal with it.

When planning care for adult patients, which oral intake is adequate to meet daily fluid needs of a stable patient? a. 500 to 1500 mL b. 1200 to 2200 mL c. 2000 to 3000 mL d. 3000 to 4000 mL

c. 2000 to 3000 mL Daily fluid intake and output is usually 2000 to 3000 mL. This is sufficient to meet the needs of the body and replace both sensible and insensible fluid losses. These would include urine output and fluids lost through the respiratory system, skin, and GI tract.

Which patient would be at greatest risk for the potential development of hypermagnesemia? a. 83-year-old man with lung cancer and hypertension b. 65-year-old woman with hypertension taking Beta-adrenergic blockers c. 42-year-old woman with systemic lupus erythematosus and renal failure d. 50-year-old man with benign prostatic hyperplasia and a urinary tract infection

c. 42-year-old woman with systemic lupus erythematosus and renal failure Rationale: Causes of hypermagnesemia include renal failure (especially if the patient is given magnesium products), excessive administration of magnesium for treatment of eclampsia, and adrenal insufficiency.

Which of the following is the most reliable way to monitor a patient's fluid status? a. I&O b. Skin turgor c. Daily weights d. Lung sounds

c. Daily weights are the best way to monitor fluid imbalances. They are easier to monitor accurately than intake and output.

The long-term care nurse is evaluating the effectiveness of protein supplements for an older resident who has a low serum total protein level. Which assessment finding indicates that the patient's condition has improved? a. Hematocrit 28% b. Absence of skin tenting c. Decreased peripheral edema d. Blood pressure 110/72 mm Hg

c. Decreased peripheral edema

The nurse is present with the physician when genetic testing results are revealed to expectant parents. Which of the following actions would be an important role of the nurse in this process? a. Realize that a positive test result will lead to elation and relief on the part of the expectant parents. b. The nurse is able to impart expertise on genetic counseling in an unlimited manner. c. Parents are tested to confirm their genotype and non-paternity may be an issue. d. Realize that a negative test result may lead to feelings of unworthiness, confusion, anger, and depression.

c. Parents are tested to confirm their genotype and non-paternity may be an issue. Rationale: Although family and individual anxiety may be decreased with a negative test result, potential problems do exist and the nurse must be prepared to address them. #1 is incorrect because negative test results would lead to elation and relief. # 2 is incorrect because nurses must be able to recognize the limits of their expertise and know how to refer a client to genetic specialists and additional resources. #4 is incorrect because it is a positive result that would elicit these feelings.

The nurse is admitting a new client, 80 years old, with congestive heart failure into your home health agency. The following assessment findings have been determined after meeting the client: overweight but no gain since the client left the hospital two days ago; VS: T 99.0, HR 100, R 22, BP 130/86. Foods eaten include canned soup at each meal, ham, and cheese. When completing the care plan for this client, the nurse should include which of the following nursing diagnosis: a. Improved Gas Exchange b. Risk for Fluid Volume Deficit c. Risk for Fluid Volume Imbalance d. Impaired Skin Integrity

c. Risk for Fluid Volume Imbalance Rationale: Sodium is found in high quantities in the foods noted that the client has consumed. When sodium levels increase in the body, water is retained, adding to the volume of fluid in circulation, making it harder for the body to move fluids through the circulation. Therefore, the excess fluid may in time impair gas exchange if levels eventually act on the lungs; fluid volume is increasing, not decreasing, in this situation, and this problem has no involvement with platelets.

The nurse should be alert for which manifestations n a patient receiving a loop diuretic? a. Restlessness and agitation b. Paresthesias and irritability c. Weak, irregular pulse and poor muscle tone d. Increased blood pressure and muscle spasms

c. Weak, irregular pulse and poor muscle tone Rationale: Loop diuretics may result in renal loss of potassium (i.e., hypokalemia). Clinical manifestations of hypokalemia include fatigue, muscle weakness, leg cramps, nausea, vomiting, paralytic ileus, soft, muscle flab, paresthesias, decreased reflexes, weak, irregular pulse, polyuria, hyperglycemia, and electrocardiographic changes.

The typical fluid replacement for the patient with a fluid volume deficit is: a. dextran b. 0.45% saline c. lactated Ringer's d. 5% dextrose in 0.45% saline

c. lactated Ringer's Rationale: Administration of an isotonic solution expands only the extracellular fluid (ECF). There is no net loss or gain from the intracellular fluid (ICF). An isotonic solution is the ideal fluid replacement for a patient with an ECF volume deficit. Examples of isotonic solutions include lactated Ringer's solution and 0.9% NaCl.

Spironolactone (Aldactone), an aldosterone antagonist, is prescribed for a patient. Which statement by the patient indicates that the teaching about this medication has been effective? a. "I will try to drink at least 8 glasses of water every day." b. "I will use a salt substitute to decrease my sodium intake." c. "I will increase my intake of potassium-containing foods." d. "I will drink apple juice instead of orange juice for breakfast."

d. "I will drink apple juice instead of orange juice for breakfast."

When planning the care of a patient with dehydration, what would the nurse instruct the unlicensed assistive personnel (UAP) to report? a. 60 mL urine output in 90 minutes b. 1200 mL urine output in 24 hours c. 300 mL urine output per 8-hour shift d. 20 mL urine output for 2 consecutive hours

d. 20 mL urine output for 2 consecutive hours The minimal urine output necessary to maintain kidney function is 30 mL/hr. If the output is less than this for 2 consecutive hours, the nurse should be notified so that additional fluid volume replacement therapy can be instituted.

The nurse incorporates the use of a pedigree when taking a family history on the client. What is an advantage of incorporating this information into the delivery of nursing care? a. A nurse's learning can be enhanced by the visual teaching contribution a pedigree can bring and also clarify any inheritance misunderstandings or misconceptions. b. It is important to gather a two-generation family pedigree even if the nurse believes this is the first occasion of the condition within a family. c. A pedigree is a word representation of the medical history of a family. d. A pedigree provides the nurse with a clear, visual representation of relationships of affected individuals to the immediate and extended family.

d. A pedigree provides the nurse with a clear, visual representation of relationships of affected individuals to the immediate and extended family. Rationale: A nurse should know how to take a family history, record the history in a pedigree, and include genetic principles. # 1 is incorrect because it is the family's learning that is enhanced. # 2 is incorrect because a pedigree should be three-generation and not two. # 3 is incorrect because a pedigree is a pictorial representation or diagram.

A patient who is taking a potassium-wasting diuretic for treatment of hypertension complains of generalized weakness. It is most appropriate for the nurse to take which action? a. Assess for facial muscle spasms. b. Ask the patient about loose stools. c. Suggest that the patient avoid orange juice with meals. d. Ask the health care provider to order a basic metabolic panel.

d. Ask the health care provider to order a basic metabolic panel.

Parents of a newborn are confused when their child is diagnosed with a genetic disorder because neither of them has a defect. Testing is done and it is determined that both parents are carriers of the disorder even though they are asymptomatic. Understanding the principles of the Mendelian Pattern of Inheritance what condition is the likely reason for this genetic disorder? a. Recessive versus dominant condition b. Autosomal dominant conditions c. X-linked recessive condition d. Autosomal recessive condition

d. Autosomal recessive condition Rationale: Knowledge of inheritance allows the nurse to not only offer and reinforce genetic information to clients and their families but also to assist them in managing their care and in making reproductive decisions. The only answer choice here that can be correct is autosomal recessive condition. In this condition the parents would be known to be carriers of the condition and they do not usually exhibit any signs or symptoms of the condition.

The nurse has been invited to discuss "the importance of promoting a good fluid and electrolyte balance in children" for a group of parents at the local school parents club meeting. Of the following actions, which is not representative of this topic? a. Recognizing possible risk factors for fluid and electrolyte balance, such as prolonged or repeated vomiting, frequent watery stools, or inability to consume fluids b. Increasing fluid intake before, during, and after strenuous exercise, particularly when the environmental temperature is high, and replacing lost electrolytes from excessive perspiration as needed with commercial electrolyte solutions c. Consuming six to eight glasses of water daily d. Encouraging excessive amounts of foods or fluids high in salt or caffeine

d. Encouraging excessive amounts of foods or fluids high in salt or caffeine Rationale: Salt causes the body to retain fluids due to an increase in the concentration of sodium and the release of ADH. Caffeine acts as a diuretic in individuals and may lead to loss of excess fluids in the body. The remaining identified measures are all appropriate.

When planning care for a patient with dehydration related to nausea and vomiting, the nurse would anticipate which fluid shift to occur because of the fluid volume deficit? a. Fluid movement from the blood vessels into the cells b. Fluid movement from the interstitial spaces into the cells c. Fluid movement from the blood vessels into interstitial spaces d. Fluid movement from the interstitial space into the blood vessels

d. Fluid movement from the interstitial space into the blood vessels In dehydration, fluid is lost first from the blood vessels. To compensate, fluid moves out of the interstitial spaces into the blood vessels to restore circulating volume in that compartment. As the interstitial spaces then become volume depleted, fluid moves out of the cells into the interstitial spaces.

When assessing a patient admitted with nausea and vomiting, which finding supports the nursing diagnosis of deficient fluid volume? a. Polyuria b. Decreased pulse c. Difficulty breathing d. General restlessness

d. General restlessness Restlessness is an early cerebral sign that dehydration has progressed to the point where an intracellular fluid shift is occurring. If the dehydration is left untreated, cerebral signs could progress to confusion and later coma.

A patient is being treated for hypokalemia. When evaluating his response to potassium replacement therpy, which of the following changes in his assessment should the nurse observe for? a. Improving visual acuity b. Worsening constipation c. Decreasing serum glucose d. Increasing muscle strength

d. Hypokalemia is associated with muscle weakness.

You are caring for an older patient who is receiving IV fluids postoperatively. During the 8:00 AM assessment of this patient, you note that the IV solution, which was ordered to infuse at 125 mL/hr, has infused 950 mL since it was hung at 4:00 AM. What is the priority nursing intervention? a. Notify the physician and complete an incident report. b. Slow the rate to keep vein open until next bag is due at noon. c. Obtain a new bag of IV solution to maintain patency of the site. d. Listen to the patient's lung sounds and assess respiratory status.

d. Listen to the patient's lung sounds and assess respiratory status. After 4 hours of infusion time, 500 mL of IV solution should have infused, not 950 mL. This patient is at risk for fluid volume excess, and you should assess the patient's respiratory status and lung sounds as the priority action and then notify the physician for further orders.

You are caring for a patient receiving calcium carbonate for the treatment of osteopenia. Which serum laboratory result would you identify as an adverse effect related to this therapy? a. Sodium falling to 138 mEq/L b. Potassium rising to 4.1 mEq/L c. Magnesium rising to 2.9 mg/dL d. Phosphorus falling to 2.1 mg/dL

d. Phosphorus falling to 2.1 mg/dL Calcium has an inverse relationship with phosphorus in the body. When phosphorus levels fall, calcium rises, and vice versa. Since hypercalcemia rarely occurs as a result of calcium intake, the patient's phosphorus falling to 2.1 mg/dL (normal 2.4-4.4 mg/dL) may be a result of the phosphate-binding effect of calcium carbonate.

Which patient is most at risk for fluid excess? a. An infant with pneumonia b. A teen with multiple injuries following and automobile accident. c. A middle-aged man who has just had surgery d. An elderly patient receiving IV therapy

d. The elderly have reduced kidney function and may not be able to handle excess fluids

The nurse is evaluating the medication list of a newly-admitted client with hypokalemia. The client has been experiencing dysrhythmias and is on a cardiac monitor. Which medication from the list would alert the nurse to pursue a discussion with the physician? a. an oral antidiabetic agent b. a cardiac rhythm drug c. an over the counter antacid d. a diuretic

d. a diuretic Excess potassium loss through the kidneys is often secondary to drugs such as potassium-wasting diuretics. Hypokalemia may lead to cardiac dysrhythmias. Potassium-wasting diuretic therapy may require supplemental oral potassium.

While assisting a new client from a wheelchair to a bed in the emergency room, the client complains of being dizzy. Which intervention by the nurse would be the best in this situation: ___________________ . a. assess peripheral pulses b. take an apical pulse c. assess for diuretic use d. assess blood pressure

d. assess blood pressure Postural or orthostatic hypotension is a sign of hypovolemia. A drop of more than 15 mmHg in systolic blood pressure when changing positions (lying to sitting, sitting to standing) often indicates fluid depletion. Assessing for diuretic use may yield information that contributes to the problem, but directly assessing the blood pressure is an immediate response to an exhibited symptom.

The nurse is unable to flush a central venous access device and suspects occlusion. The best nursing intervention would be to: a. apply warm moist compresses to the insertion site b. attempt to force 10mL of normal saline into the device c. place the patient on the left side with head-down position d. instruct the patient to change positions, raise arm, and cough.

d. instruct the patient to change positions, raise arm, and cough. Rationale: Interventions for catheter occlusion include instructing the patient to change position, raise an arm, and cough; assessing for and alleviating clamping or kinking of the tube; flushing the catheter with normal saline through a 10-mL syringe (do not force flush); using fluoroscopy to determine cause and site of occlusion; and instilling anticoagulant or thrombolytic agents.

A patient has the following arterial blood gas results: pH 7.52; PaCO2 30 mmHg; HCO3- 24 mEq/L. The nurse determines that these results indicate: a. metabolic acidosis b. metabolic alkalosis c. respiratory acidosis d. respiratory alkalosis

d. respiratory alkalosis Rationale: Respiratory alkalosis (carbonic acid deficit) occurs with hyperventilation. The primary cause of respiratory alkalosis is hypoxemia from acute pulmonary disorders. Anxiety, central nervous system (CNS) disorders, and mechanical overventilation also increase ventilation rate and decrease the partial pressure of arterial carbon dioxide (PaCO2). This leads to a decrease in carbonic acid level and to alkalosis.

During the postoperative care of a 76-year-old patient, the nurse monitors the patient's intake and output carefully, knowing that the patient is at risk for fluid and electrolyte imbalances primarily because: a. older adults have an impaired thirst mechanism and need reminding to drink fluids b. water accounts for a greater percentage of body weight in the older adult than in younger adults c. older adults are more likely than younger adults to lose extracellular fluid during surgical procedures d. small losses of fluid are more significant because body fluids account for only about 50% of body weight in older adults

d. small losses of fluid are more significant because body fluids account for only about 50% of body weight in older adults Rationale: In the older adult, body water content averages 45% to 55% of body weight.


Related study sets

Foundations exam 2 review questions

View Set

The Magician's Nephew Chapters 13-15

View Set

Beginner Korean Grammar Sentence Practice (Part 1/2)

View Set

Business Finance - Some Lessons from Capital Market History

View Set